LSAT and Law School Admissions Forum

Get expert LSAT preparation and law school admissions advice from PowerScore Test Preparation.

User avatar
 Dave Killoran
PowerScore Staff
  • PowerScore Staff
  • Posts: 5852
  • Joined: Mar 25, 2011
|
#82506
Complete Question Explanation
(The complete setup for this game can be found here: https://forum.powerscore.com/lsat/viewtopic.php?t=14240)

The correct answer choice is (C)

Answer choice (A) is incorrect because, as discussed in the setup, M and S must always play the same sport.

Answer choice (B) is incorrect because from the last rule when P plays tennis then K plays tennis.

Answer choice (C) is the correct answer.

Answer choice (D) is incorrect because from the last rule when P plays tennis he must rank lower than O.

Answer choice (E) is incorrect because from the fifth rule when M and S play tennis, S must rank higher than M.

Get the most out of your LSAT Prep Plus subscription.

Analyze and track your performance with our Testing and Analytics Package.